Tag Archives: doituyen

Đáp án đề thi chọn đội tuyển trường PTNK năm 2020

Ngày thi thứ nhất.

Bài 1. Với mỗi số nguyên dương $n$, tìm số thực $M_{n}$ lớn nhất sao cho với mọi số thực dương $x_{1}, x_{2}, \ldots, x_{n}$ thì ta đều có
$$
\sum_{k=1}^{n} \frac{1}{x_{k}^{2}}+\frac{1}{\left(\sum_{k=1}^{n} x_{k}\right)^{2}} \geq M_{n}\left(\sum_{k=1}^{n} \frac{1}{x_{k}}+\frac{1}{\sum_{k=1}^{n} x_{k}}\right)^{2}
$$

Bài 2. Cho 2021 số nguyên khác 0 . Biết rằng tổng của một số bất kỳ trong chúng với tích của tất cả 2020 số còn lại luôn âm.
(a) Chứng minh rằng với mọi cách chia 2021 số này thành hai nhóm và nhân các số cùng nhóm lại với nhau thì tổng của hai tích cũng luôn âm.
(b) Một bộ số thỏa mãn đề bài thì có thể có nhiều nhất mấy số âm?

Bài 3. Cho hai hàm số $f: \mathbb{R} \rightarrow \mathbb{R}$ và $g: \mathbb{R} \rightarrow \mathbb{R}$ thỏa mãn $g(2020)>0$ và với mọi $x, y \in \mathbb{R}$ thì $\left\{\begin{array}{l}f(x-g(y))=f(-x+2 g(y))+x g(y)-6 \\ g(y)=g(2 f(x)-y)\end{array}\right.$

(a) Chứng minh rằng $g$ là hàm hằng.

(b) Chứng minh rằng đồ thị $h(x)=f(x)-x$ nhận $x=1$ là trục đối xứng.

Bài 4. Cho tam giác $A B C$ nhọn, nội tiếp trong đường tròn $(O)$ có trực tâm $H$ và $A H, B H, C H$ cắt cạnh đối diện lần lượt tại $D, E, F$. Gọi $I, M, N$ lần lượt là trung điểm các cạnh $B C, H B, H C$ và $B H, C H$ cắt lại $(O)$ theo thứ tự tại các diểm $L, K$. Giả sử $K L$ cắt $M N$ ở $G$.
(a) Trên $E F$, lấy điểm $T$ sao cho $A T$ vuông góc với $H I$. Chứng minh rằng $G T$ vuông góc với $O H$.
(b) Gọi $P, Q$ lần lượt là giao điểm của $D E, D F$ và $M N$. Gọi $S$ là giao điểm của $B Q, C P$. Chứng minh rằng $H S$ di qua trung điểm của $E F$.

Ngày thi thứ hai.
Bài 5. Cho số nguyên dương $n>1$. Chứng minh rằng với mọi số thực $a \in\left(0 ; \frac{1}{n}\right)$ và mọi đa thức $P(x)$ có bậc $2 n-1$ thỏa mãn điều kiện $P(0)=P(1)=0$, luôn tồn tại các số thực $x_{1}, x_{2}$ thuộc $[0 ; 1]$ sao cho $P\left(x_{1}\right)=P\left(x_{2}\right)$ và $x_{2}-x_{1}=a$.

Bài 6. Giải phương trình sau trên $\mathbb{Z}^{+}:\left(x^{2}+3\right)^{3^{x+1}}\left[\left(x^{2}+3\right)^{3^{x+1}}+1\right]+x^{2}+y=x^{2} y$.

Bài 7 . Cho các số nguyên $n>k>t>0$ và $X={1,2, \ldots, n}$. Gọi $\mathcal{F}$ là họ các tập con có $k$ phần tử của tập hợp $X$ sao cho với mọi $F, F^{\prime} \in \mathcal{F}$ thì $\left|F \cap F^{\prime}\right| \geq t$. Giả sử không có tập con có $t$ phần tử nào chứa trong tất cả các tập $F \in \mathcal{F}$.
(a) Chứng minh rằng tồn tại một tập hợp $B \subset X$ sao cho $|B|<3 k$ và $|B \cap F| \geq t+1$ với mọi $F \in \mathcal{F}$.
(b) Chứng minh rằng $|\mathcal{F}|<C_{3 k}^{t+1} C_{n}^{k-t-1}$.

Bài 8. Cho tam giác $A B C$ nội tiếp trong $(O)$ với $B, C$ cố định và $A$ thay đổi trên cung lớn $B C$. Dựng hình bình hành $A B D C$ và $A D$ cắt lại $(B C D)$ ở $K$.
(a) Gọi $R_{1}, R_{2}$ lần lượt là bán kính đường tròn ngoại tiếp $(K A B),(K A C)$. Chứng minh rằng tích $R_{1} R_{2}$ không đổi.
(b) Ký hiệu $(T),\left(T^{\prime}\right)$ lần lượt là các đường tròn cùng đi qua $K$, tiếp xúc với $B D$ ở $B$ và tiếp xúc với $C D$ ở $C$. Giả sử $(T),\left(T^{\prime}\right)$ cắt nhau ở $L \neq K$. Chứng minh rằng $A L$ luôn đi qua một điểm cố định.

Hết

Đáp án thi chọn Đội Tuyển Trường PTNK năm học 2013-2014

Đề thi và đáp án kì thi chọn đội tuyển Toán trường Phổ thông Năng khiếu – ĐHQG TPHCM được tổ chức vào tháng 10 năm 2013, chọn ra 6 học sinh dự thi kì thi HSG Quốc gia năm 2014. Các thí sinh từ các lớp 11, 12 (chủ yếu là học sinh chuyên toán), thực hiện bài thi trong 2 ngày, mỗi ngày 4 bài, mỗi bài 180 phút. Sau đây là đề thi và đáp án thực hiện bởi Star Education.

Ngày thi thứ 1

Bài 1. Tìm tất cả các hàm số $f: \mathbb{R} \rightarrow \mathbb{R}$ thoả mãn

$$f(x^{3}+y+f(y))=2 y+x^{2} f(x), \forall x, y \in \mathbb{R}$$

Bài 2. Cho dãy $\left\{u_{n}\right\}$ thoả mãn $u_{1}=2013, u_{n+1}=u_{n}^{3}-4 u_{n}^{2}+5 u_{n} \forall n \in \mathbb{N}^{*}$. Tìm tất cả các số nguyên tố $p$ là ước của $\left(u_{2014}+2009\right)$ và $p \equiv 3(\bmod 4)$.

Bài 3. Trong một hội nghị khoa học có 5000 đại biểu tham dự, mỗi một đại biểu biết ít nhất một thứ tiếng. Một uỷ ban gồm một số đại biểu được gọi là “uỷ ban làm viẹc” nếu tất cả thành viên trong uỷ ban đều biết chung một thú tiếng; gọi là “uỷ ban thách thức” nếu không có hai thành viên nào của uỷ ban biết chung một thứ tiếng (uỷ ban có thểgồm 1 thành viên; uỷ ban này gọi là làm việc họ̆c thách thức đều được). Chứng minh rằng có thể chia các đại biểu thành 100 uỷ ban rời nhau (mỗi đại biểu thuộc một uỷ ban) sao cho các uỷ ban này họ̆c là uỷ ban làm việc hoặc là uỷ ban thách thức.

Bài 4. Tam giác $A B C$ có $B, C$ cố định còn $A$ di động sao cho $A B=A C$ và $\angle B A C>60^{\circ} .$ Đường thẳng đối xúng với $B C$ qua $A B$ cắt AC tai $P$. Trên đoạn $P C$ lấy $M$ sao cho $P M=P B$. Gọi $N$ là giao điểm của $A B$ với phân giác ngoài góc BCA. Chứng minh $M N$ luôn đi qua một điểm cố định.

Ngày thi thứ 2

Bài 5. Cho 2014 số thực $x_{1}, x_{2}, \ldots, x_{2014}$ thỏa mãn điều kiện $\sum_{i=1}^{2014} x_{i}=0$ và $\sum_{i=1}^{2014} x_{i}^{2}=2014$. Tìm giá trị lớn nhất của biểu thức $P=x_{1} x_{2} \cdots x_{2014}$.

Bài 6. Cho dãy số $u_{n}$ xác định bởi $u_{1}=1, u_{n+1}=\frac{u_{n}}{\sqrt{u_{n}^{2}+1}+\sqrt{2}}$ với mọi $n \in \mathbb{N}^{*}$. Tìm giới hạn $\lim \frac{u_{n+1}}{u_{n}}$.

Bài 7. Cho n nguyên dương và A là tập con khác rỗng của $X={1,2, \ldots, n}$.

  1. Tính giá trị của tổng $S(A)=\sum_{E C X} \cdot(-1)^{|E \cap A|}$,trong đó $E$ lấy trên tất cả các tập con của tập $X$ (kể cả tập rỗng).

  2. Cho $m \in \mathbb{N}^{*}$,xét $m$ tập con khác rỗng của $X$ là $A_{1}, A_{2}, \ldots, A_{m}$ và $m$ số nguyên khác không là $a_{1}, a_{2}, \ldots, a_{m}$ sao cho $a_{1}+a_{2}+\cdots+a_{m}<0$. Chứng minh tồn tại tập con $E$ của $X$ sao cho $\sum_{i=1}^{m}(-1)^{|E \cap A|} a_{i}>0$ (Kí hiệu $|A|$ chỉ số phần tử của tập $A$, số phần tử của tập rỗng là 0 ).

Bài 8. Cho tam giác $A B C$ nhọn có $H$ là trực tâm và $P$ là điểm di động bên trong tam giác $A B C$ sao cho $\angle B P C=\angle B H C$. Đường thẳng qua $B$ và vuông góc với $A B$ cắtPC tại $M$.Đường thẳng qua $C$ và vuông góc với $A C$ cắt $P B$ tại N. Chứng minh rằng trung điểm I của $M N$ luôn thuộc một đường cố định.

Hết

Giải

Bài 1.

Trong phương trình đã cho, thay $x=y=0$, ta có $f(f(0))=0$. \medskip

Lại thay $y=0$ thì $$f(f^3+f(0))=x^2f(x), \, \forall x.$$

Thay $y=f(0)$ thì $$f(x^3+f(0))=2f(0)+x^2f(x).$$

Từ đây suy ra $f(0)=0$. Thay $y=0$ vào đẳng thức đã cho ta được $f(x^3)=x^2f(x)$. Do đó ta có $$f(x^3+y+f(y))=2y+f(x^3) \text{ hay } f(x+y+f(y))=2y+f(x). \eqno{(*)}$$
Thay $y$ bởi $-y$, ta được $$f(x-y+f(-y))=-2y+f(x).$$
Với $x$ bất kì, ta lấy $2y=f(x)$ ta được $f(x-y+f(-y))=0$ suy ra $x-y+f(-y)=0$. Do đó, ta được $f(-x)=f(-y+f(-y))=-2y=-f(x).$
Từ đây suy ra
$$f(x+f(y)+f(f(y)))=2f(y)+f(x).$$
Trong $(*)$ thay $x=-y$ ta được $f(f(y))=2y+f(-y)=2y-f(y)$, kết hợp với đẳng thức trên, ta được $$f(x+2y)=2f(y)+f(x).$$ Đến đây cho $x=0$ ta được $f(2y)=2f(y)$ nên ta được $f(x+y)=f(x)+f(y)$, tức là $f(x)$ cộng tính.
Đến đây ta sẽ tính $f((x+1)^3+(x-1)^3)$ theo hai cách như sau

  • $f((x+1)^3+(x-1)^3)=f(2x^3+6x)=2x^2f(x)+6f(x).$
  • $f((x+1)^3+(x-1)^3)=(x+1)^2f(x+1)+(x-1)^2f(x-1)=(x+1)^2(f(x)+f(1))+(x-1)^2(f(x)-f(1))=2x^2f(x)+2f(x)+4xf(1).$

So sánh hai đẳng thức trên, ta được $f(x)=xf(1)=ax$ với mọi $x$. Thử lại ta được $a=1, a=-2$. \medskip

Vậy các hàm cần tìm là $f(x)=x, f(x)=-2x$.

Bài 2.

Ta có
$$\begin{aligned} u_{n+1}-2 & =(u_n-2)(u_{n-1}-1)^2 \\
& = (u_{n-2}-1)^2(u_{n-1}-1)^2(u_{n-2}-2) \\
&= (u_{n-1}-1)^2(u_{n-2}-1)^2 \cdots (u_2-1)^2(u_1-2). \end{aligned} $$

Do đó $$u_{2014}+2009= 2011 \left[ (u_{2013}-1)^2(u_{2012}-1)^2 \cdots (u_2-1)^2 +1 \right].$$

Gọi $B$ là biểu thức trong dấu ngoặc vuông thứ hai. Ta có bổ đề quen thuộc là nếu $a^2+b^2$ chia hết cho số nguyên tố $p=4k+3$ thì $a,b$ cùng chia hết cho $p.$ Từ đây suy ra số $B$ có dạng $a^2+1$ nên nó sẽ không có ước nguyên tố dạng $4k+3$. \medskip

Vậy $u_{2014}+9$ chỉ có một ước nguyên tố $p \equiv 3 \pmod{4}$ duy nhất là $2011$.

Bài 3. Trước hết, ta chứng minh bổ đề sau \medskip

Định lý Ramsey Với $s,t$ là các số nguyên dương, gọi $R(s,t)$ là số đỉnh ít nhất cần có của một graph để trong đó luôn tồn tại một tập độc lập $s$ đỉnh hoặc một graph con đầy đủ $t$ đỉnh. Khi đó
$$R(s,t)\le C_{s+t-2}^{s-1}. \eqno{(*)}$$

Chứng minh
Ta sẽ chứng minh rằng $$R(s,t)\le R(s-1,t)+R(s,t-1).$$
Để ý rằng với $s=1$ hoặc $t=1$ thì $R(s,t)=1$. Do đó, nếu chứng minh được đánh giá này thì chỉ cần dùng tính chất của tam giác Pascal để có $$R(s,t)\le C_{s+t-3}^{s-2}+C_{s+t-3}^{s-1}=C_{s+t-2}^{s-1}.$$
Đặt $n$ là vế phải của (*) và xét graph $G$ có $n$ đỉnh. Xét $v\in G$ thì

  • Nếu như có ít nhất $R(s,t-1)$ đỉnh kề với $v$. Khi đó, theo định nghĩa thì trong tập đỉnh đó, sẽ luôn có một tập độc lập $s$ đỉnh hoặc một graph con đầy đủ $t-1$ đỉnh, ghép thêm đỉnh $v$ vào thì thỏa mãn điều kiện của $R(s,t).$
  • Nếu như có ít nhất $R(s-1,t)$ đỉnh không kề với $v$. Tương tự trên, trong tập đỉnh đó, cũng sẽ có một một graph con đầy đủ $t$ đỉnh hoặc tập độc lập $s-1$ đỉnh, ghép thêm đỉnh $v$ vào thì thỏa mãn điều kiện của $R(s,t).$

    Từ đó, ta thấy graph $G$ này thỏa mãn điều kiện của $R(s,t)$ nên theo tính nhỏ nhất thì $R(s,t)\le n.$

Trở lại bài toán, \medskip

Xét graph đơn vô hướng $G=(V,E)$ đại diện cho hội nghị khoa học đã nêu, trong đó $V$ là tập hợp các đại biểu và hai đỉnh được nối nhau nếu hai đại biểu tương ứng quen nhau. Ta gọi $T$ là tập hợp đỉnh biểu diễn cho thành viên của ban tổ chức. \medskip

Khi đó một ủy ban gồm $5$ thành viên là đại diện nếu như đó là một graph đầy đủ, còn đó là thách thức nếu đó là graph không có cạnh. Ta gọi các graph con như thế là graph con “chuẩn”. \medskip

Trong các đỉnh $V\backslash T,$ ta xóa dần dần các graph con chuẩn đến khi không thực hiện được nữa. Ta gọi tập hợp còn lại là $S.$ Ta sẽ chứng minh rằng $S\cup T$ có thể phân hoạch thành các graph con chuẩn như trên. \medskip

Theo định lý Ramsey, rõ ràng $|S| \le C_{8}^{4}=70$. Xét một đỉnh $v \in S$ thì giả thiết, $v$ kề với cả $280$ đỉnh của $T$ nên ta chọn ra trong đó $4$ đỉnh để ghép với $v$ tạo thành một graph con “chuẩn”. Cứ như thế thực hiện cho đến hết các phần tử của $S$, còn lại bao nhiêu phần tử trong $T$ thì chia đều ra thành các graph con “chuẩn” là được. \medskip

Bài toán được giải quyết.

Bài 4.

Tam giác $PBM$ cân tại $P$ nên bằng biến đổi góc, ta có

$$\angle{PBM}=\angle{PMB} \Rightarrow 2\angle{ABC}-\angle{MBC}= \angle{ACB}+\angle{MBC}.$$

Do đó $\angle{ABC}=2\angle{MBC}$ nên $BM$ là tia phân giác của $\angle{ABC}.$ Theo tính chất đường phân giác thì
$$\frac{MC}{MA}=\frac{BC}{BA}=\frac{BC}{AC}.$$

Lại có $CN$ là phân giác ngoài của $\angle{ACB}$ nên ta cũng có
$\frac{NA}{NB}=\frac{CA}{CB}.$ Gọi $I$ là trung điểm của $BC$ thì $I$ là điểm cố định. \medskip

Xét tam giác $ABC$ với $I$ thuộc $BC$ , $M$ thuộc $AC$ và $N$ thuộc $AB$ với

$$\frac{IB}{IC} \cdot \frac{MC}{MA} \cdot \frac{NA}{NB}=1 \cdot \frac{BC}{AC} \cdot \frac{AC}{BC}=1$$

thì theo định lý Menelaus đảo, ta có $M , N , I$ thẳng hàng. \medskip

Vậy $MN$ luôn đi qua điểm $I$ cố định.

Bài 5. 

Rõ ràng có thể chọn giá trị các biến thích hợp để $P>0$ nên để tìm giá trị lớn nhất của $P$ thì ta chỉ xét các số ${{x}_{1}},{{x}_{2}},\ldots ,{{x}_{2014}}$ đều khác $0$ và số các số âm là chẵn. Không mất tính tổng quát, giả sử
${{x}_{1}}\ge {{x}_{2}}\ge \ldots \ge {{x}_{2m}}>0>{{x}_{2m+1}}\ge \ldots \ge {{x}_{2014}}.$
Đổi dấu các số ${{y}_{k}}=-{{x}_{k}}>0$ với $2m+1\le k\le 2014.$ Khi đó ta viết lại
$$\left\{ \begin{aligned}
& {{x}_{1}}+{{x}_{2}}+\cdots +{{x}_{2m}}={{y}_{1}}+{{y}_{2}}+\cdots +{{y}_{2n}}=A \\
& x_{1}^{2}+x_{2}^{2}+\cdots +x_{2m}^{2}+y_{1}^{2}+y_{2}^{2}+\cdots +y_{2n}^{2}=2014 \\
\end{aligned} \right.$$
trong đó $m+n=1007$ (ngoài ra $m,n>0$ vì các số đã cho không thể toàn bộ là dương hoặc toàn bộ là âm). Theo bất đẳng thức Cauchy-Schwarz thì
$$2014\ge \frac{{{A}^{2}}}{2m}+\frac{{{A}^{2}}}{2n} \text{ nên } {{A}^{2}}\le 4mn.$$
Lại theo bất đẳng thức AM-GM thì
$$\begin{aligned} P& =({{x}_{1}}{{x}_{2}}\ldots {{x}_{2m}})({{y}_{1}}{{y}_{2}}\ldots {{y}_{2n}})\le {{\left( \frac{A}{2m} \right)}^{2m}}{{\left( \frac{A}{2n} \right)}^{2n}} \\
&=\frac{{{A}^{2m+2n}}}{{{2}^{2m+2n}}{{m}^{2m}}{{n}^{2n}}}\le \frac{{{(4mn)}^{m+n}}}{{{2}^{2m+2n}}{{m}^{2m}}{{n}^{2n}}}={{\left( \frac{m}{n} \right)}^{n-m}}. \end{aligned}$$

Do $m,n$ khác tính chẵn lẻ nên với vai trò bình đẳng của $m,n,$ ta có thể giả sử $m<n$ nên $n-m\ge 1$ và $m\le 503.$ Khi đó, áp dụng bất đẳng thức Bernoulli thì

$${{\left( \frac{n}{m} \right)}^{n-m}}\ge 1+\left( \frac{n}{m}-1 \right)(n-m)=1+\frac{{{(n-m)}^{2}}}{m}\ge 1+\frac{1}{503}=\frac{504}{503}.$$
Suy ra $P\le {{\left( \frac{m}{n} \right)}^{n-m}}\le \frac{503}{504}.$ Đây chính là giá trị lớn nhất cần tìm, dấu bằng xảy ra khi
$m=503,n=504$ và $${{x}_{1}}={{x}_{2}}=\cdots ={{x}_{1006}}=\sqrt{\frac{504}{503}},{{x}_{1007}}={{x}_{1008}}=\cdots ={{x}_{2014}}=-\sqrt{\frac{503}{504}}.$$

Bài 6.

Xét hàm số $f(x)=\frac{x}{\sqrt{{{x}^{2}}+1}+\sqrt{2}}$ với $x\in \mathbb{R}$ thì $${f}'(x)=\frac{1+\sqrt{2+2{{x}^{2}}}}{\sqrt{1+{{x}^{2}}}{{\left( \sqrt{2}+\sqrt{1+{{x}^{2}}} \right)}^{2}}}>0$$ nên hàm này đồng biến trên $\mathbb{R}.$
Dãy số đã cho được viết lại thành
$$\left\{ \begin{aligned}
& {{u}_{1}}=1, \\
& {{u}_{n+1}}=f({{u}_{n}}),n\ge 1 \\
\end{aligned} \right.$$ thì ${{u}_{1}}<{{u}_{2}}$ nên dễ dàng chứng minh quy nạp được rằng dãy này giảm. \medskip

Do dãy này bị chặn dưới bởi $0$ nên nó có giới hạn, đặt giới hạn đó là $L\ge 0$. Trong biểu thức xác định dãy, cho $n\to +\infty ,$ ta được $$L=\frac{L}{\sqrt{{{L}^{2}}+1}+\sqrt{2}}$$ nên $L=0.$
Từ đó suy ra
$$\underset{n\to +\infty }{\mathop{\lim }}\,\frac{{{u}_{n+1}}}{{{u}_{n}}}=\underset{n\to +\infty }{\mathop{\lim }}\,\frac{1}{\sqrt{u_{n}^{2}+1}+\sqrt{2}}=\frac{1}{1+\sqrt{2}}.$$

Bài 7.

(a) Nếu $A=X$ thì $$S(A)=\sum\limits_{E\subset X}(-1)^{|E|}=C_n^0-C_n^1+C_n^2-\cdots +(-1)^nC_n^n=0.$$

Còn nếu $A\neq X$, do $S(A)$ chỉ phụ thuộc vào số phần tử của $A$ nên không mất tính tổng quát, ta giả sử rằng $A=\{1,2,\ldots ,k\}$ với $k<n$. Khi đó, ta có
$$\begin{aligned} S(A) & =\sum_{E\subset X-\{k\}}(-1)^{|E\cap A|}+\sum_{E\subset X-\{k\}}(-1)^{|(E\cup\{k\})\cap A|} \\
& =\sum_{E\subset X-\{k\}}(-1)^{|E\cap A|}-\sum_{E\subset X-\{k\}}(-1)^{|E\cap A|}=0. \end{aligned} $$
Vậy $S(A)=0,\forall A\subset X$. \medskip

(b) Đặt $f(E)=\sum_{i=1}^{m}(-1)^{|E\bigcap A_i|}a_i$. Giả sử $f(E)\leq 0, \, \forall E$. Mà ta cũng có
$$\sum_{E\subset X}f(E)=\sum_{i=1}^ma_iS(A_i)=0.$$
Suy ra $f(E)=0,\, \forall E \subset X$, nhưng điều này là không thể vì $f(\varnothing)<0$. Vậy luôn tồn tại $E$ sao cho $f(E)>0$.

 

Bài 8. 

Vẽ đường kính $AA’$ của đường tròn $(ABC)$. Vì $A’B \perp AB$ nên $B,A’,M$ thẳng hàng. Tương tự thì $C,A’,N$ thẳng hàng. Giả sử
$BA’, CA’$ cắt lại $(BHC)$ lần lượt tại $E,F$. Mặt khác

$$\angle NPM=180{}^\circ -\angle BHC=\angle A=180{}^\circ -\angle B{A}’C=\angle M{A}’N$$

nên $PA’MN$ là tứ giác nội tiếp.

Ta sẽ chứng minh trung điểm của $A’F, A’E, MN$ là thẳng hàng. Theo định lý Menelaus đảo thì điều nào tương đương với $$ \dfrac{\overline{A’F}}{\overline{A’N}} = \dfrac{\overline{EA’}}{\overline{EM}} \Leftrightarrow \dfrac{\overline{A’F}}{\overline{A’E}} = – \dfrac{\overline{A’N}}{\overline{EM}} \Leftrightarrow \dfrac{A’B}{A’C} = \dfrac{A’N}{ME}. \eqno{(*)}$$

Vì $\angle BN{A}’=\angle CME$ và $\angle NB{A}’=\angle MCE$ nên hai tam giác $BN{A}’,CME$ đồng dạng với nhau. Do đó
$\frac{{A}’N}{ME}=\frac{{A}’B}{CE}$.
Mặt khác, bằng biến đổi góc, ta cũng có $C{A}’E$ cân tại $C$ nên $CE=C{A}’.$ Ta có được $$\frac{{A}’N}{ME}=\frac{{A}’B}{{A}’C}.$$
Do đó, khẳng định $(*)$ là đúng. Vậy nên điểm $I$ luôn nằm trên đường trung bình của tam giác $A’EF$ là đường cố định.

Bạn đọc có thể tìm thêm nhiều cách giải cho bài 8 này tại

link sau

Tham khảo từ sách “Tuyển tập đề thi môn Toán đội tuyển và dự tuyển trường PTNK”

Đáp án thi chọn Đội Tuyển thi Quốc Gia của trường PTNK năm học 2015 – 2016

Ngày thứ 1

Bài 1. Cho tập hợp
$$
A=\{n \in \mathbb{N} \mid 1 \leq n \leq 2015,(n, 2016)=1\}
$$
Hỏi có bao nhiêu số nguyên $a \in A$ sao cho tồn tại số nguyên b mà $a+2016 b$ là số chính phương?

Bài 2. Cho $a, b, c, d$ là các số thực thỏa mãn điều kiện
$$
a^{2} \leq 1, a^{2}+b^{2} \leq 5, a^{2}+b^{2}+c^{2} \leq 14, a^{2}+b^{2}+c^{2}+d^{2} \leq 30
$$
1. Chúng minh rằng $a+b+c+d \leq 10$.
2. Chứng minh rằng $a d+b c \leq 10$.

Bài 3. Tìm tất cả các hàm số $f: \mathbb{R} \rightarrow \mathbb{R}$ thỏa mãn điều kiện
$$
f(x-2 f(y))=5 f(x)-4 x-2 f(y)
$$
với mọi $x, y \in \mathbb{R}$.

Bài 4. Cho đường tròn $k$ và các điểm $B, C$ thuộc đường tròn, không phải là đường kính; I là trung điểm $B C$. Điểm $A$ di động trên cung lớn $B C$ của $k$. Gọi $i_{1}$ là đường tròn qua $I$ và tiếp xúc với $A B$ tại $B ; i_{2}$ là đường tròn qua $I$ và tiếp xúc với $A C$ tại $C$. Các đường tròn $i_{1}, i_{2}$ cắt nhau tại $D$ (khác $I$ ).
1. Chứng minh rằng đường tròn ngoại tiếp tam giác AID luôn đi qua một điểm cố định.
2. Gọi $K$ là trung điểm $A D$, $E$ là tâm đường tròn qua $K$ và tiếp xúc với $A B$ tại $A, F$ là tâm đường tròn qua $K$ và tiếp xúc với AC tại $A$. Chứng minh rằng góc EAF có số đo không đổi.

Ngày thứ 2

Bài 5. Dãy số $\left(x_{n}\right)$ được xác định bởi công thức $x_{n}=\frac{1}{n \cos \frac{1}{n}}$ với mọi $n \geq 1$. Tính giới hạn sau
$$\lim \frac{x_{1}+x_{3}+x_{5}+\cdots+x_{2 n-1}}{x_{2}+x_{4}+x_{6}+s+x_{2 n}}$$

Bài 6. Tim các giá trị của $b$ sao cho tồn tại a để hệ phương trình sau có nghiệm $(x, y)$
$$
\left\{\begin{array}{l}
(x-1)^{2}+(y+1)^{2}=b \\y=x^{2}+(2 a+1) x+a^{2}
\end{array}\right.
$$

Bài 7. Cho n là số nguyên dương, $n \geq 2$ và $X={1,2,3, \ldots, n}$. Gọi $A_{1}, A_{2}, \ldots, A_{m}$ và $B_{1}, B_{2}, \ldots, B_{m}$ là hai dãy các tập con khác rỗng của $X$ thỏa mãn điều kiện: Với mỗi $i, j \in{1,2,3, \ldots, n}, A_{i} \cap B_{j}=\varnothing$ nếu và chỉ nếu $i=j$.
1. Chúng minh rằng với mỗi hoán vị $\left(x_{1}, x_{2}, \ldots, x_{n}\right)$ của $X$, có không quá một cặp tập hợp $\left(A_{i}, B_{i}\right)$ với $i=1,2,3, \ldots, n$ sao cho nếu $x_{k} \in A_{i}$ và $x_{l} \in B_{i}$ thì $k<l$.
2. Gọi $a_{i}, b_{i}$ lần lượt là số phần tử của tập hợp $A_{i}, B_{i}$ với $i=1,2,3, \ldots, m$. Chúng minh rằng
$$
\sum_{i=1}^{m} \frac{1}{C_{a_{i}+b_{i}}^{a_{i}}} \leq 1
$$

Bài 8. Cho tam giác $A B C$ nhọn nội tiếp đường tròn tâm $O$. Đường tròn tâm $I$ đi qua $B$, $C$ lần lượt cắt các tia $B A$, CA tại $E, F$.
1. Giả sử các tia $B F, C E$ cắt nhau tại $D$ và $T$ là tâm đường tròn $(A E F)$. Chứng minh rằng $O T$ || ID.
2. Trên BF, CE lần lượt lấy các điểm $G, H$ sao cho $A G \perp C E, A H \perp B F$. Các đường tròn $(A B F),(A C E)$ cắt $B C$ tai $M, N$ (khác $B, C)$ và cắt EF tại $P, Q$ (khác $E, F)$. Gọi $K$ là giao điểm của $M P, N Q$. Chứng minh rằng DK vuông góc với GH.

Giải

Bài 1.

Cho $n$ là số nguyên dương lớn hơn 1 , ta quy ước gọi một số nguyên dương a được gọi là thặng dư chính phương theo modulo $n$ nếu $(a, n)=1$ và tồn tại số nguyên $x$ sao cho $a \equiv x^{2}(\bmod n)$. Trong bài này, dể dơn giản, ta quy ước xét các thặng dư chính phưong nhỏ hơn $n$.
Đặt $s(n)$ là số các số nhỏ hơn $n$ và là thặng dư chính phương theo modulo n. Ta sễ chứng minh hai bổ dề dưới đây:
Bổ đề 1: Cho $p$ là số nguyên tố và $k$ là số nguyên dưong. Khi đó:
1. Nếu $p=2$ thì $s\left(2^{k}\right)=2^{\max (k-3,0)}$.
2. Nếu $p>2$ thì $s\left(p^{k}\right)=\frac{p^{k}-p^{k-1}}{2}$.
Bổ đề $2: s(n)$ là hàm nhân tính.
Thật vậy,
Trước hết, ta biết rằng $s(p)=\frac{p-1}{2}$ với $p$ là số nguyên tố lẻ. Ta sẽ tính $s\left(p^{k}\right)$ với $k \in \mathbb{Z}^{+}$.
Xét một thặng dư chính phương $a$ của $p$, khi đó tồn tại $x$ sao cho
$$
a \equiv x^{2}(\bmod p)
$$
Dặt $a=x^{2}+p q$ thì hiển nhiên
$$
a \equiv x^{2}+p q\left(\bmod p^{k}\right) \Leftrightarrow a-p q \equiv x^{2}\left(\bmod p^{k}\right)
$$
và khi đó, ta có $p^{k-1}$ cách chọn $q$ để các số $a-p q$ là các thặng dư chính phưong $\bmod p^{k}$.
Suy ra
$$
s\left(p^{k}\right)=p^{k-1} s(p)=\frac{p^{k}-p^{k-1}}{2}
$$
Xét số nguyên tố $p=2$, với $k=1,2,3$, dể dàng kiểm tra được $s\left(2^{k}\right)=1$.
Ta xét $k \geq 4$, tưong tự trên, ờ bước chọn $q$, ta chỉ có 2 cách nên $s\left(2^{k}\right)=$ $2 s\left(2^{k-1}\right)$. Từ đó bằng quy nạp, ta có được
$$
s\left(2^{k}\right)=2^{k-3}, k \geq 4
$$
Tiếp theo, xét hai số $a, b$ nguyên dương và $(a, b)=1 .$ Gọi $A$ là tập họp các thặng dư chính phương theo modulo $a b$ và $B$ là tập hợp các số là thặng dư chính phưong chung của $a, b$.
Nếu $x \in A$ thì tổn tại $y$ sao cho $x \equiv y^{2}(\bmod a b)$. Rō ràng khi đó,
$$
x \equiv y^{2} \quad(\bmod a), x \equiv y^{2} \quad(\bmod b)
$$

(chú ý rằng nếu $x>a$, ta có thể chọn $x^{\prime}$ sao cho $x^{\prime}<a$ và $x \equiv x^{\prime}(\bmod a)$; tương tự với $b$ ). Do đó, $x \in B$, tức là $x \in A \Rightarrow x \in B$ nên $|A| \leq|B|$.
Tiếp theo, xét $x \in B$. Khi đó tồn tại $r, s$ sao cho $x \equiv r^{2}(\bmod a), x \equiv s^{2}$ $(\bmod b)$. Theo định lý thặng dư Trung Hoa, tổn tại số nguyên $z$ sao cho
$$
z \equiv r(\bmod a), z \equiv s(\bmod b)
$$
Khi đó
$$
x \equiv z^{2} \quad(\bmod a), x \equiv z^{2} \quad(\bmod b)
$$
nên
$$
x-z^{2}: a b \text { hay } x \equiv z^{2}(\bmod a b)
$$
Do đó: $x \in A$, tức là $x \in B \Rightarrow x \in A$ nên $|A| \geq|B|$.
Từ đây ta có
$$
|A|=|B| \text { hay } s(a) s(b)=s(a b)
$$
Vậy $s(n)$ là hàm nhân tính.
Các bổ đề đều được chứng minh.
Trở lại bài toán, ta thấy rằng
$$
2016=2^{5} \cdot 3^{2} \cdot 7
$$
Rō ràng bài toán yêu cầu đếm số thặng dư chính phương theo modulo 2016. Theo bổ dề 2 thì
$$
s(2016)=s\left(2^{5}\right) s\left(3^{2}\right) s(7)
$$
Theo bổ đề 1 thì
$$
s\left(2^{5}\right)=2^{2}=4, s\left(3^{2}\right)=\frac{3^{2}-3}{2}=3, s(7)=\frac{7-1}{2}=3
$$
Do đó, số các số $a$ cần tìm là $4 \cdot 3 \cdot 3=36$.

Bài 2.

1) Dự đoán dấu bằng xảy ra khi $a=1, b=2, c=3, d=4$ nên ta có các đánh giá sau
$$
\left\{\begin{array}{l}
a^{2}+1 \geq 2 a \\
b^{2}+4 \geq 4 b \\
c^{2}+9 \geq 6 c \\
d^{2}+16 \geq 8 d
\end{array}\right.
$$
Do đó, ta có
$$
\begin{aligned}
&24(a+b+c+d) \leq 3\left(d^{2}+16\right)+4\left(c^{2}+9\right)+6\left(b^{2}+4\right)+12\left(a^{2}+1\right) \\
&=3 d^{2}+4 c^{2}+6 b^{2}+12 a^{2}+120 \\
&=3\left(a^{2}+b^{2}+c^{2}+d^{2}\right)+\left(a^{2}+b^{2}+c^{2}\right)+2\left(a^{2}+b^{2}\right)+6 a^{2}+120 \\
&\leq 3 \cdot 30+14+2 \cdot 5+6 \cdot 1+120=240
\end{aligned}
$$
Suy ra $a+b+c+d \leq 10$.
2) Ta có:
$$
16 a^{2}+d^{2} \geq 8 a d \text { và } 9 b^{2}+4 c^{2} \geq 12 b c
$$
Từ đó suy ra
$$
\begin{aligned}
&24(a d+b c) \leq 3\left(16 a^{2}+d^{2}\right)+2\left(9 b^{2}+4 c^{2}\right) \\
&=3\left(a^{2}+b^{2}+c^{2}+d^{2}\right)+5\left(a^{2}+b^{2}+c^{2}\right)+10\left(a^{2}+b^{2}\right)+30 a^{2} \\
&\leq 3 \cdot 30+5 \cdot 14+10 \cdot 5+30 \cdot 1=240
\end{aligned}
$$
Suy ra $a d+b c \leq 10$.

Bài 3.

Goi $(*)$ là điều kiện đề bài cho. Trong $(*)$, thay $x=y=0$, ta có
$$
f(-2 f(0))=3 f(0)
$$
Đặt $f(0)=a$ thì $f(-2 a)=3 a$. Trong $(*)$, thay $x=0$ và $y=-2 a$, ta có
$$
f(-2 f(-2 a))=5 a-2 f(-2 a) \Leftrightarrow f(-6 a)=-a
$$

Trong $(*)$, thay $x=-2 a, y=-6 a$, ta có
$$
\begin{aligned}
&f(-2 a-2 f(-6 a))=5 f(-2 a)-4 x-2 f(-6 a) \\
&\Leftrightarrow f(0)=15 a+8 a+2 a \\
&\Leftrightarrow a=25 a \\
&\Leftrightarrow a=0
\end{aligned}
$$
Do đó $f(0)=0$.
Trong $(*)$, thay $y=0$, ta có
$$
f(x)=5 f(x)-4 x \Leftrightarrow f(x)=x
$$
Thử lại ta thấy thỏa.
Vậy hàm số cần tìm chính là
$$
f(x)=x, \forall x \in \mathbb{R}
$$

Bài 4.

1) Gọi $O$ là tâm của đường tròn $k$. Không mât tính tống quát, giả sử tia $\Lambda D$ nằm giữa hai tia $A O, A B$, các trường hợp còn lại tương tự.
Ta có:
$$
\angle I D B=\angle A B C, \angle I D C=\angle A C B
$$
nên
$$
\angle B A C+\angle B D C=\angle B A C+\angle A B C+\angle A C B=180^{\circ}
$$

Do đó, tứ giác $A B D C$ nội tiếp hay $D \in(O)$. Ta thấy
$$
\begin{aligned}
&\angle D A O+\angle O I D \\
&=\angle B A C-(\angle D A B+\angle O A C)+360^{\circ}-\left(90^{\circ}+\angle D I C\right) \\
&=\angle B A C-\left(\angle I C D+90^{\circ}-\angle A B C\right)+270^{\circ}-\angle D I C \\
&=\angle B A C+\angle A B C-(\angle I C D+\angle D I C)+180^{\circ} \\
&=\left(180^{\circ}-\angle A C B\right)-\left(180^{\circ}-\angle I D C\right)+180^{\circ} \\
&=\angle I D C-\angle A C B+180^{\circ}=180^{\circ}
\end{aligned}
$$

Do đó, AOID nội tiếp hay đường tròn $(A I D)$ di qua $O$ cố định.
2) Ta có:
$$
\angle E A C=90^{\circ}-\angle B A C, \angle F A B=90^{\circ}-\angle B A C
$$
nên
$$
\angle E A F=180^{\circ}-2 \angle B A C+\angle B A C=180^{\circ}-\angle B A C
$$
Do đó, góc $\angle E A F$ có số đo không đổi.

Bài 5.

Trước hết, ta chứng minh bổ đề sau:
Giá trị của biểu thức
$$
\frac{1}{1}+\frac{1}{2}+\frac{1}{3}+\cdots+\frac{1}{n}
$$
tiến tới vô cực khi $n \rightarrow+\infty$. Thật vậy,
Xét hàm số $f(x)=\ln (1+x)-x$ với $x>0$. Ta có
$$
f^{\prime}(x)=\frac{1}{1+x}-1<0
$$
nên đây là hàm nghịch biến, suy ra $f(x)<f(0)=0$ hay $\ln (1+x)<$ $x, \forall x>0$. Thay $x$ bởi $\frac{1}{n}$, ta được
$$
\ln \left(1+\frac{1}{n}\right)<\frac{1}{n} \Leftrightarrow \frac{1}{n}>\ln (1+n)-\ln n
$$
Do đó,
$$
\frac{1}{1}+\frac{1}{2}+\frac{1}{3}++\frac{1}{n}>\ln 2-\ln 1+\ln 3-\ln 2+\cdots+\ln (n+1)-\ln n=\ln (n+1)
$$
Vì $\ln (n+1) \rightarrow+\infty$ khi $n \rightarrow+\infty$ nên
$$
\frac{1}{1}+\frac{1}{2}+\frac{1}{3}+\cdots+\frac{1}{n} \rightarrow+\infty
$$
Trở lại bài toán, đặt
$$
y_{n}=\frac{x_{1}+x_{3}+x_{5}+\cdots+x_{2 n-1}}{x_{2}+x_{4}+x_{6}+\cdots+x_{2 n}}
$$
với $n \geq 1$. Ta thấy vì $\frac{1}{n} \in\left(0 ; \frac{\pi}{2}\right)$ nên $\cos \frac{1}{n}>0$, suy ra
$$
x_{n}=\frac{1}{n \cos \frac{1}{n}}>0, n \geq 1
$$

Xét hàm số $f(t)=\frac{t}{\cos t}$ với $t \in\left(0 ; \frac{\pi}{2}\right)$ thì $f^{\prime}(t)=\frac{\cos t+t \sin t}{\cos ^{2} t}>0$ nên đây là hàm đồng biến. Chú ý rằng $x_{n}=f^{2}\left(\frac{1}{n}\right)$, mà $\frac{1}{n}$ là dãy giảm nên $x_{n}$ cũng là dãy giảm.
Suy ra $x_{1}>x_{2}, x_{3}>x_{4}, \ldots, x_{2 n-1}>x_{2 n}$ nên $y_{n}>1$
Ngoài ra, ta cũng có $x_{3}<x_{2}, x_{5}<x_{4}, \ldots, x_{2 n-1}<x_{2 n-2}$ nên
$$
\begin{aligned}
y_{n}<& \frac{x_{1}+\left(x_{2}+x_{4}+\cdots+x_{2 n-2}\right)}{x_{2}+x_{4}+\cdots+x_{2 n}}=\\
& 1-\frac{x_{1}-x_{2 n}}{x_{2}+x_{4}+\cdots+x_{2 n}}<1-\frac{x_{1}}{x_{2}+x_{4}+\cdots+x_{2 n}}
\end{aligned}
$$
Dễ thấy rằng
$$
x_{2}+x_{4}+\cdots+x_{2 n}=\sum_{i=1}^{n} \frac{1}{2 i \cos \frac{1}{2 i}} \geq \sum_{i=1}^{n} \frac{1}{2 i}=\frac{1}{2} \sum_{i=1}^{n} \frac{1}{i}
$$
Theo bổ đề trên thì $\sum_{i=1}^{n} \frac{1}{i}$ tiến tới vô cực nên
$$
\lim \left(x_{2}+x_{4}+\cdots+x_{2 n}\right)=+\infty
$$
Do dó
$$
\lim \left(1-\frac{x_{1}}{x_{2}+x_{4}+\cdots+x_{2 n}}\right)=1-0=1
$$
Theo nguyên lý kẹp, ta có $\lim x_{n}=1$.

Bài 6.

Đặt $X=x-1, Y=y+1$, thay vào, ta có
$$
\begin{aligned}
&\left\{\begin{array}{l}
X^{2}+Y^{2}=b \\
Y-1=(X+1)^{2}+(2 a+1)(X+1)+a^{2}
\end{array}\right. \\
&\Leftrightarrow\left\{\begin{array}{l}
X^{2}+Y^{2}=b \\
Y=X^{2}+(2 a+3) X+a^{2}+2 a+3
\end{array}\right.
\end{aligned}
$$
Ta đưa về tìm điều kiện của $b$ để tồn tại $a$ mà hệ trên có nghiệm $(X, Y)$. Do
$$
Y-(X+2)=X^{2}+2(a+1) X+(a+1)^{2}=(X+a+1)^{2} \geq 0
$$

nên $Y \geq X+2$. Suy ra $Y-X \geq 2>0$, tức là $(X-Y)^{2} \geq 4$. Ta có
$$
b=X^{2}+Y^{2}=\frac{(X-Y)^{2}+(X+Y)^{2}}{2} \geq \frac{(Y-X)^{2}}{2} \geq 2
$$
Mặt khác, với $b \geq 2$, nếu chọn $X=-(a+1)$ thì có $Y=X+2=1-a$. Khi đó, ta có
$$
X^{2}+Y^{2}=(a+1)^{2}+(a-1)^{2}=2\left(a^{2}+1\right)=b
$$
Như thế, với $a$ thỏa mãn $2\left(a^{2}+1\right)=b$ thì hệ có nghiệm là
$$
(X, Y)=(-a-1,1-a)
$$
Dễ dàng thấy rằng do $b \geq 2$ nên luôn tồn tại $a$ như thế.
Vậy các giá trị cần tìm của $b$ là $b \geq 2$.

Bài 7.

1) Giả sử ngược lại, tồn tại 2 cặp $\left(A_{i}, B_{i}\right)$ và $\left(A_{j}, B_{j}\right)$ thỏa mãn điểu kiện đề bài đã cho.
Vì $i \neq j$ nên theo giả thiết,
$$
\left|A_{i} \cap B_{j}\right| \geq 1,\left|A_{j} \cap B_{i}\right| \geq 1
$$
Đặt $x_{r} \in A_{i} \cap B_{j}, x_{s} \in A_{j} \cap B_{i}$ với $1 \leq r, s \leq n$ thì:
– Do $x_{r} \in B_{j}$ nên với mọi $x_{k} \in A_{j}$, ta đều có $k<r$.
– Do $x_{r} \in A_{i}$ nên với mọi $x_{k} \in B_{i}$, ta đều có $k>r$.

Từ đây suy ra
$$
A_{j} \subset\left\{x_{1}, x_{2}, \ldots, x_{r-1}\right\}, B_{i} \subset\left\{x_{r+1}, x_{r+2}, \ldots, x_{n}\right\}
$$
Điều này cho thấy $A_{j} \cap B_{i}=\varnothing$, mâu thuẫn với giả thiết.
Vậy tồn tại không quá 1 cặp $\left(A_{i}, B_{i}\right)$ thỏa mãn điều kiện đã cho.
2) Gọi $T$ là tập hợp các cách chọn hai dãy
$$
A_{1}, A_{2}, \ldots, A_{m} \text { và } B_{1}, B_{2}, \ldots, B_{m}
$$
thỏa mãn điều kiện là: với mỗi $i, j \in\{1,2,3, \ldots, n\}, A_{i} \cap B_{j}=\varnothing$ nếu và chỉ nếu $i=j$.
Gọi $T_{i} \subset T$ là các cách chọn sao cho sao cho cặp $\left(A_{i}, B_{i}\right)$ thỏa mãn điều kiện là: cặp $\left(A_{i}, B_{i}\right)$ với $i=1,2,3, \ldots, n$ sao cho nếu $x_{k} \in A_{i}$ và $x_{l} \in B_{i}$ thì $x_{k}<x_{l}$ (ở đây ta xét thứ tự ban đầu của các phần tử của $X$ ). (*)
Theo câu 1) thì $T_{i} \cap T_{j}=\varnothing$ với $i \neq j$ nên ta có
$$
\left|T_{1}\right|+\left|T_{2}\right|+\cdots+\left|T_{m}\right|=\left|T_{1} \cup T_{2} \cup \ldots \cup T_{m}\right| \leq T
$$
Tiếp theo, với $1 \leq i \leq m$, xét một tập hợp $S \subset X$ và $|S|=a_{i}+b_{i}$. Khi đó, tương ứng với $S$, có đúng 1 cách chọn $\left(A_{i}, B_{i}\right)$ thỏa mãn tính chất $(*)$ – tức là $A_{i}$ sẽ nhận $a_{i}$ số nhỏ nhất trong tập $S, B_{i}$ là lấy phần còn lại.
Trong khi đó, nếu không có điều kiện $(*)$, ta có thể chọn tùy ý $C_{a_{i}+b_{i}}^{a_{i}}$ phần tử trong $S$ và $A$ và số còn lại cho $B$.
Do đó, ta có
$$
\left|T_{i}\right|=\frac{|T|}{C_{a_{i}}^{a_{i}}+b_{i}}
$$
với $i=1,2, \ldots, m$. Từ đây suy ra
$$
\sum_{i=1}^{m} \frac{|T|}{C_{a_{i}+b_{i}}^{a_{i}}} \leq|T| \Leftrightarrow \sum_{i=1}^{m} \frac{1}{C_{a_{i}+b_{i}}^{a_{i}}} \leq 1
$$
Ta có đpcm.

Bài 8.

1) Giả sử $E F$ cắt $B C$ ở $L$ và $(T),(O)$ cắt nhau tại $J$ khác $A$. Suy ra $A J$ chính là trục đẳng phương của $(T),(O)$. Do đó $O T \perp A J$.
Khi đó,
$$
L B \cdot L C=L E \cdot L F
$$
nên $L$ thuộc trục đẳng phương của $(T),(O)$. Suy ra $A, J, L$ thẳng hàng. Theo định lý Brocard cho tứ giác $B E F C$ nội tiếp trong đường tròn $(I)$ thì $I$ chính là trực tâm của tam giác $A D L$.
Vì thế nên ID $\perp A L$, mà $O T \perp A J$ nên $I D \| O T$.

2) Dễ dàng thấy rằng $D$ là trực tâm của tam giác $A G H$ nên $A D \perp G H$. Ta sẽ chứng minh rằng $A, D, K$ thẳng hàng.

Ta có $D B \cdot D F=D E \cdot D C$ nên $D$ có cùng phương tích tới 2 đường tròn $(A B F),(A E C)$. Suy ra $A D$ chính là trục đẳng phương của 2 đường tròn này.

Bằng biến đổi các góc nội tiếp, ta thấy rằng
$$
\angle M P Q=\angle M B F=\angle C E F=\angle C N Q
$$
Suy ra $M N P Q$ nội tiếp, dẫn đến $K M \cdot K P=K N \cdot K Q$, tức là $K$ cũng có cùng phương tích tới 2 đường tròn $(A B F),(A E C)$.
Từ đó suy ra $A, D, K$ thẳng hàng. Do đó, $D K$ vuông góc với $G H$.